1. Trang chủ
  2. » Giáo Dục - Đào Tạo

Chuyên đề bất đẳng thức xoay vong nguyễn văn cường

66 1 0

Đang tải... (xem toàn văn)

Tài liệu hạn chế xem trước, để xem đầy đủ mời bạn chọn Tải xuống

THÔNG TIN TÀI LIỆU

Nội dung

Trang 1

Lời mở đầu

Trong bất đẳng thức cổ điển thì bất đẳng thức xoay vịng là một nội dung hay vàkhó Có những bất đẳng thức có dạng khá đơn giản nhưng phải mất hàng chục năm,nhiều nhà tốn học mới giải quyết được Ví dụ như bất đẳng thức Shapiro được đặtra vào năm 1903 bởi Neishbitt.

Với 3 số không âm a, b, c chứng minh rằng:ab + c+bc + a +ca + b ≥ 32 (đơn giản)và dạng tổng quát:Mở rộng với n số a1, a2, , an thì:a1a2+ a3 +a2a3+ a4 + · · · +ana1+ a2 ≥ n2Khì nào đúng, khi nào sai.

Đến năm 1954 tức là sau 52 năm, Shapiro mới tổng kết lại giả thuyết này nhưsau:

1) Bất đằng thức đúng với n lẻ ≤ 232) Bất đằng thức đúng với n chẵn ≤ 12Cịn lại sai.

Hồn tồn tự nhiên ta thấy cịn rất nhiều dạng bất đẳng thức xoay vịng khácthì bất đẳng thức là gì, khi nào đúng, khi nào sai hoặc luôn luôn đúng Trong bài luậnvăn này chúng tôi xây dựng được một dạng bất đẳng thức xoay vòng tổng quát màcác trường hợp riêng là những bài tốn khó và rất khó có thể sử dụng trong những đềthi học sinh giỏi.

Luận văn này gồm có 2 chương:

Chương 1: Bất đẳng thức xoay vịng (Trình bày những kết quả đã có vềcác bài bất đẳng thức phân thức.)

Trang 3

1 Bất đẳng thức xoay vòng 4

1.1 Bất đẳng thức Schurs 4

1.1.1 Bất đẳng thức Schurs và hệ quả 4

1.1.2 Một số bài toán minh họa 9

1.2 Bất đẳng thức xoay vòng khác trong tam giác 12

1.3 Sử dụng bất đẳng thức Cauchy chứng minh một số dạng bất đẳng thứcxoay vòng 23

1.4 Bất đẳng thức xoay vòng phân thức 32

2 Một dạng bất đẳng thức xoay vòng 412.1 Các trường hợp đơn giản 41

2.1.1 Trường hợp 3 số n = 3 412.1.2 Trường hợp 4 số n = 4 422.1.3 Trường hợp 5 số n = 5 432.1.4 Trường hợp 6 số n = 6 452.1.5 Trường hợp 7 số n = 7 472.2 Trường hợp tổng quát 53

2.2.1 Một số kiến thức liên quan 53

2.2.2 Nhận xét đặc biệt 53

Trang 4

Bất đẳng thức xoay vòng

1.1Bất đẳng thức Schurs

1.1.1Bất đẳng thức Schurs và hệ quả

Bài 1 (Bất đẳng thức Schurs)

Với x, y, z là các số thực dương, λ là một số thực bất kì, chứng minh rằng:

xλ(x − y)(x − z) + yλ(y − z)(y − x) + zλ(z − x)(z − y) ≥ 0

Dấu bằng xảy ra khi vào chỉ khi x = y = zChứng minh

Chú ý rằng khi có hai biến số bằng nhau thì bất đẳng thức hiển nhiên đúng.Chẳng hạn khi y = z ta có: xλ(x − z)2 ≥ 0 Dấu ” = ” xảy ra khi x = y = z Khơngmất tính tổng qt ta có thể giả thiết rằng: x > y > z

+ Xét trường hợp λ ≥ 0

Bất đẳng thức có thể viết lại dưới dạng:

(x − y)[xλ(x − z) + yλ(y − z)] + zλ(z − x)(z − y) ≥ 0

Sử dụng điều kiện x > y ta thu được

Trang 5

do đó bất đẳng thức đúng.+ Xét trường hợp λ < 0Ta có

M = xλ(x − y)(x − z) + (y − z)[zλ(x − z) − yλ(x − y)]

Sử dụng điều kiện y > z (hay x − z > y − z ) ta có:

M > xλ(x − y)(x − z) + (y − z)(x − y)(zλ− yλ) > 0, (∀λ < 0)

Vậy bất đẳng thức cần được chứng minh.Bài 2 (Bất đẳng thức Schurs mởi rộng)

Giả sử I là một khoảng thuộc R và f : I −→ R+ là một hàm đơn điệu hayf ”(x) ≥ 0, ∀x ∈ I Với x1, x2, x3 ∈ I, chứng minh rằng:

f (x1)(x1− x2)(x1− x3) + f (x2)(x2− x3)(x2− x1) + f (x3)(x3− x1)(x3− x2) ≥ 0 (1)

Dấu ” = ” xảy ra khi và chỉ khi x1 = x2 = x3.Chứng minh

Vì f là hàm đơn điệu hay f ”(x) ≥ 0, x ∈ I nên ta có bất đẳng thức:f [λx + (1 − λ)y] < f (x)

λ +

f (y)

1 − λ (2)

∀x, y ∈ I và λ ∈ (0, 1)

Khơng mất tính tổng qt ta giả sử x1 < x2 < x3 (vì nếu 2 trong 3 biến bằngnhau thì bất đẳng thức ln đúng, dấu bất đẳng thức xảy ra khi x1 = x2 = x3 ).

Chia hai vế của (1) cho (x2− x3)(x2− x1) < 0 ta thu được:

Trang 6

Đặt: λ = x3− x2x3− x1 ⇒1 − λ = x2− x1x3− x1x2 = λx1+ (1 − λ)x3

ta thu được bất đẳng thức (2) đúng hay (1) đúng.Bài 3 (Một dạng mở rộng của bất đẳng thức Schurs)

Xét a, b, c, u, c, w là các số thực dương chứng minh rằng:a) Nếu p > 0 và

a1p + cp1 ≤ b1p; u1+p1 + w1+p1 ≥ v1+p1

Ta có: ubc − vca + wab ≥ 0b) Nếu −1 < p < 0 và

a1p + cp1 ≤ b1p; u1+p1 + w1+p1 ≤ v1+p1

Ta có: ubc − vca + wab ≤ 0c) Nếu p < −1

a1p + cp1 ≥ b1p; u1+p1 + w1+p1 ≤ v1+p1

Ta có: ubc − vca + wab ≥ 0

Dấu bằng của bất đẳng thức xảy ra khi và chỉ khi

ap1 + c1p = b1p; u1+p1 + w1+p1 = v1+p1Chứng minha) Nếu p > 0 ta có: 11 + p +1p+1p= 1Áp dụng bất đẳng thức Holder ta có:a1+p1 (uc)1+p1 + c1+p1 (wa)1+p1 ≤a1p + c1pp+1p(uc + wa)p+11

Trang 7

Áp dụng giả thiết bài tốn ta có:

acv ≤ acu1+p1 + w1+p1

p+1

≤ b(uc + wa)

suy ra ubc − acv + wab ≥ 0b) Với −1 < p < 0 ta cũng có:1p + 1 +1p+1p= 1 với p + 1p < 0

Khi đó bất đẳng thức Holder có chiều ngược lại:

a1+p1 (uc)1+p1 + c1+p1 (wa)1+p1 ≥a1p + c1p

p+1p

(uc + wa)p+11

Lũy thừa p + 1 hai vế ta được

⇔ acu1+p1 + w1+p1p+1≥a1p + c1pp+1(uc + wa)Áp dụng giả thiết phần b) (chú ý p + 1 > 0, p < 0) ta có:acv ≥ acu1+p1 + w1+p1p+1≥ (uc + wa)a1p + c1pp≥ (uc + wa)b

suy ra: abw − auv + ubc ≤ 0c) Với p < −1 ta cũng có:1p + 1 +1p+1p= 1 với p + 1 < 0Áp dụng bất đẳng thức Holder:a1+p1 (uc)1+p1 + c1+p1 (wa)1+p1 ≤a1p + c1pp+1p(uc + wa)p+11

Lũy thừa p + 1 hai vế (chú ý p + 1 > 0) ta được:

acu1+p1 + w1+p1

p+1

≤a1p + c1p

p

Trang 8

Áp dụng giả thiết phần c) (chú ý p + 1 < 0) ta có:acv ≤ acu1+p1 + w1+p1p+1≤ (uc + wa)a1p + c1pp≤ (uc + wa)b

suy ra: ucb − acv + wab ≥ 0Bài 4 (Bài toán hệ quả 1)

Với x > y > z > 0 f là hàm đơn điệu hay f ”(x) = 0 ∀x > 0 và f nhận giá trịtrên R+, chứng minh rằng:f (x)y − z +f (y)z − x +f (z)x − y ≥ 0Chứng minhÁp dụng bài tốn 2 ta có:

f (x)(x − y)(x − z) + f (y)(y − z)(y − x) + f (z)(z − x)(z − y) ≥ 0

Chia 2 vế cho (y − z)(z − x)(x − y) < 0 ta thu được bất đẳng thức cần chứngminh.

Bài 5 (Bài toán hệ quả 2)

Với x, y, z, a, b, c > 0 thỏa mãn điều kiện:

a2+ b2 ≤ c2x23 + y23 ≥ z23chứng minh rằng xa +yb ≥ zcChứng minh

Áp dụng bài tốn 3 với p = 1

2 ta có:

xbc − zab + yac ≥ 0

Trang 9

1.1.2Một số bài tốn minh họa

Bài 6

Giả sử ∆ABC khơng nhọn, chứng minh rằng:27sin A+64sin B ≥ 125sin CChứng minh

Áp dụng bài toán 5 với điều kiện





sin2A + sinB ≤ sin2C Tam giác không nhọn2723 + 6423 = 12523

Ta thu được điều phải chứng minh.Bài 7

Cho a, b, c là các số thực dương thỏa mãn điều kiện1a3 + 1b3 + 1c3, x32 + y32 ≤ z32Chứng minh rằng: xa +yb ≤ zcChứng minhTa có:11 −13 +11−13−13= 1Áp dụng bất đẳng thức Holder ta có:a32(xb)32 + b32(ya)32 ≥ (xb + ya)32(a−3+ b−3)−12⇔ (ab)32x32 + y32≥ (xb + ya)32  1a3 + 1b3−12

Trang 10

Do đó ta có bất đẳng thức(abz)32 ≥ (ab)32x32 + y32≥ (xb + ya)32 c32⇔ abz ≥ (xb + ya)c⇔ xa +yb ≤ zcBài 8

Với a, b, c là ba cạnh của một tam giác bất kì p = a + b + c

2 , chứng minh rằng(p − a)4+ (p − b)4+ (p − c)4+ S2 ≥ a (p − a)3+ b (p − c)3+ c (p − a)3

(Với S là diện tích tam giác ABC )Chứng minh

Chứng minh bất đẳng thức Schurs với λ = 2 ta có:x2(x − y)(x − z) + y2(y − z)(y − x) + z2(z − x)(z − y) ≥ 0⇔ x4+ y4+ z4+ xyz(x + y + z) ≥ x3(y + z) + y3(z + x) + z3(x + y) (1)Đặt:x = p − ay = p − bz = p − c⇒x + y + z = p − a + p − b + p − c = pxyz = (p − a)(p − b)(p − c) = Spy + z = (p − b) + (p − c) = ax + z = b, x + y = c

Thay vào (1) ta có bất đẳng thức cần chứng minh.Bài 9

Với x, y, z dương thỏa mãn:yzx2 +zx

y2 +xyz2 = 3hãy tìm giá trị lớn nhất của biểu thức sau:

Trang 11

⇔ 3 − y + zx +z + xy +x + yz +yzx +zxy +xyz≥ 0⇔ M = y + zx +z + xy +x + yz ≤ 6Đẳng thức xảy ra khi ⇔ x = y = z = 1Vậy Mmax = 6Bài 10

Với ha, hb, hclà độ dài các đường cao của một tam giác ABC bất kì, chứng minhrằng:2 1h3a+ 1h3b+ 1h3c+ 3hahbhc ≥ 1r 1h2a+ 1h2b+ 1h2c

Trong đó r là bán kính vịng trịn nội tiếp ∆ABCChứng minh

Áp dụng bất đẳng thức Schurs với λ = 1 ta có:x(x − y)(x − z) + y(y − x)(y − z) + z(z − x)(z − y) ≥ 0⇔ x3+ y3+ z3− x2(z + y) − y2(z + x) − z2(x + y) + 3xyz⇔ 2(x3+ y3+ z3) − (x + y + z)(x2+ y2+ z2) + 3xyz ≥ 0Đặt x = 1ha, y =1hb, z =1hc (vì1ha +1hb +1hc =1r)Bài 11

Với a, b, c là ba số thực lớn hơn 1, chứng minh rằng:

Trang 12

1.2Bất đẳng thức xoay vòng khác trong tam giác

Trong mục này ta chỉ đề cập đến cách xây dựng bất đẳng thức xoay vòng trong∆ABC với 3 cặp biến quay vịng: A, B, C là 3 góc tam giác ABC và x, y, z (x, y, z là3 số thực) bắt đầu từ biểu thức luôn đúng ∀A.B, C, x, y, z

Bài 1

Với mọi ∆ABC, x, y, z là ba số thực dương tùy ý, chứng minh rằng:

yz cos A + zx cos B + xy cos C ≤ 12(x

2+ y2+ z2)

Chứng minhTa có:

(x − y cos C − z cos B)2+ (y sin C − z sin B)2 ≥ 0 ∀x, y, z > 0⇔ x2+ y2(cos2C + sin2C) + z2(cos2B + sin2B)

+ 2yz(cos B cos C − sin B sin C) − 2xy cos C − 2xz cos B ≥ 0⇔ x2+ y2+ z2− 2(yz cos A + zx cos B + xy cos C) ≥ 0

⇔ yz cos A + zx cos B + xy cos C ≤ 12(x

2+ y2+ z2)Dấu bằng xảy ra khi và chỉ khi

y cos C + z cos B = xy sin C − z sin B = 0⇔

y2cos2C + 2yz cos B cos C + z2cos2B = x2

Trang 13

Vậy:x = ka

y = kb (k > 0) ⇒ x, y, z là 3 cạnh của một tam giácz = kc đồng dạng với∆ABC

Từ bài toán 1 ta có thể xây dựng được các bất đẳng thức mới trong tam giác.Bài 2

Với mọi tam giác ∆ABC, a, b, c là 3 số thực dương, chứng minh rằng:

x cos A + y cos B + z cos C ≤ 12 xyz +yzx +zxyChứng minh

Áp dụng bài toán 1 Thay x, y, z lần lượt bởi 1x,1y,1z ta có:1yz cos A +1zxcos B +1xycos C ≤12 1x2 + 1y2 + 1z2

⇔ x cos A + y cos B + z cos C ≤ 12 yzx +zxy +xyz

Dấu bằng xảy ra khi 1x,

1y,

1

z là độ dài 3 cạnh của tam giác đồng dạng với tamgiác ABC Cho x, y, z là các giá trị cụ thể ta thu được các bài tốn tìm giá trị lớnnhất, nhỏ nhất, các bất đẳng thức khó trong tam giác.

Bài 3

Tìm giá trị lớn nhất:

M = 2 cos A + 3 cos B + 4 cos C

Trong đó A, B, C là ba góc của một tam giác.Chứng minh

Áp dụng bài toán 2 với:x = 2y = 3z = 4Ta có: M ≤ 12 4.32 +2.43 +2.34= 6112

Trang 14

là 12,13,14 Vậy max M =6112Bài 4

Cho tam giác ∆ABC, chứng minh rằng

2 sinA2 + 3 sinB2 + 4 sinC2 ≤ 6112 (1)Chứng minhĐặt:A = π − 2A0B = π − 2B0C = π − 2C0⇒ A0+ B0+ C0 = π ⇒ A0, B0, C0 là 3 góc của ∆A0B0C0

Ta có: (1) ⇔ 2 cos A0+ 3 cos B0 + 4 cos C0 ≤ 6112

Áp dụng bài tốn 3 có bất đẳng thức đúng Dấu đẳng thức xảy ra nếu

∆A0B0C0 ∼ ∆(12,12,14)Bài 5Chứng minh rằng12 sin A +13 sin B +14 sin C ≥ 10861(Trong đó A, B, C là ba góc của một tam giác nhọn)

Chứng minhÁp dụng bất đẳng thức Cauchy ta có:12 cos A +13 cos B +14 cos C

(2 cos A + 3 cos B + 4 cos C) ≥ 9 ∀∆ABC nhọn

⇔ 12 cos A +13 cos B +14 cos C ≥ 9

2 cos A + 3 cos B + 4 cos Cmà theo bài 3 ta có:

Trang 15

suy ra điều phải chứng minh12 cos A +13 cos B +14 cos C ≥ 9.1261 =10861Bài 6

Chứng minh rằng trong mọi tam giác ∆ABC ta đều có:14tan2 A2 +16tan2 B2 +16tan2 C2 ≥ 3954056 (1)Chứng minhTa có:(1) ⇔ 141cos2 A2− 1!+161cos2 B2− 1!+181cos2 C2− 1!≥ 3954056⇔ 14 cos2 A2+ 16 cos2 B2+ 18 cos2 C2≥ 108169⇔ 12 + 2 cos A +13 + 3 cos B +14 + 4 cos C ≥ 108169 (2)Áp dụng bất đẳng thức Cauchy ta có:12 + 2 cos A +13 + 3 cos B +14 + 4 cos C[(2 + 2 cos A) + (3 + 3 cos B)+ (4 + 4 cos C)] ≥ 9⇔ 12 + 2 cos A+13 + 3 cos B +14 + 4 cos C ≥ 9

2 cos A + 3 cos B + 4 cos C + 9 (3)Áp dụng kết quả bài tốn 3 ta có:

9

2 cos A + 3 cos B + 4 cos C + 9 ≥ 619

12+ 9 =108

169 (4)

Từ (3) và (4) ta có bất đẳng thức (2) đúng suy ra (1) đúng.Bài 7

Chứng minh rằng với tam giác ∆ABC nhọn ta có:a) √32 cos A +√33 cos B +√34 cos C ≤ 3r 61336b)1 + 12 cos A 1 + 13 cos B 1 + 14 cos C≥ 97613Chứng minha) Ta có:3√2 cos A +√33 cos B +√34 cos C3 ≤ 3r

Trang 16

(Chứng minh nhờ bất đẳng thức Jensen xét hàm f (t) = t13 trong (0, +∞) )Áp dụng bài toán 3 ta có:

3

r

2 cos A + 3 cos B + 4 cos C

3 ≤ 3r6136⇔√3 2 cos A +√33 cos B +√34 cos C ≥ 33r6136b) Ta cóM =1 + 12 cos A 1 + 13 cos B 1 + 14 cos C= 1+12 cos A +13 cos B +14 cos C+16 cos A cos B +112 cos B cos C +18 cos C cos A+ 1

(2 cos A)(3 cos B)(4 cos C)

Áp dụng bất đẳng thức Cauchy ta cóM ≥ 1 + 33

r

1

(2 cos A)(3 cos B)(4 cos C)+ 3

3

s

1

(2 cos A)(3 cos B)(4 cos C)2+3r1

(2 cos A)(3 cos B)(4 cos C)3

⇔ M ≥ 1 + 3 1

p(2 cos A)(3B)(4 cos C)!3

Áp dụng bất đẳng thức Cauchy ta có:

M ≥

1 + 3

2 cos A + 3 cos B + 4 cos C3≥1 + 613123=1 + 36613= 97613Bài 8

Chứng minh rằng với mọi tam giác ∆ABC ta có:a)r 116tan4 A2 +136tan4 B2 +r 136tan4 B2 +164tan4 C2+r 164tan4 C2 +116tan4 A2 ≥√2 3954056b) 3 cos B42A +4 cos C9 cos2B +2 cos A16 cos2C ≥ 10861 .Với ∆ABC là tam giác nhọn.

Chứng minh

a) Áp dụng bất đẳng thức Bunhiacopxki ta dễ dàng chứng minh được:√

a2+ b2+√

b2+ c2+√

c2+ a2√

Trang 17

Thay a = 14tan2A2, b =16tan2 B2, c =18tan2 C2Khi đó ta có:r 116tan4 A2 +136tan4 B2 +r 136tan4 B2 +164tan4 C2 +r 164tan4 C2 +136tan4 A2≥√2 14tan2 A2 +16tan2 B2 +18tan2 C2(1)Áp dụng bài tốn 6 ta có:√2 14tan2 A2 +16tan2 B2 +18tan2 C2≥ 395√24056 (2)Từ (1) và (2) ta có bất đẳng thức cần chứng minh.b) Áp dụng bất đẳng thức Bunhiacopxki ta có bất đẳng thức sau:a2b +b2c +c2a ≥ a + b + c ∀a, b, c dương (3)(Vì a2b +b2c +c2a = a2b +b2c +c2a (a + b + c)a + b + c ≥ (a + b + c)2a + b + c = a + b + c)Áp dụng (3) với a = 12 cos A, b =13 cos B, c =14 cos C Ta có:3 cos B4 cos2A +4 cos C9 cos2B +2 cos A16 cos2C ≥ 12A +13 cos B +14 cos CÁp dụng kết quả bài toán 5 ta có bất đẳng thức được chứng minh.Bài 9

Chứng minh rằng với mọi tam giác ∆ABC ta có: a) hacos A + hbcos B +hccos C ≤ 9

4R

b) (hacos A + hbcos B + hccos C)3 ≥ 27

8 hahbhc

(Với a, b, c là các đường cao tương ứng với 3 cạnh a, b, c Dấu đẳng thức xảy ra khi∆ABC đều).

Chứng minh

a) Áp dụng bài tốn 2 ta có:

Trang 18

mà 12 hahbhc+ hbhcha+hchahb= (a2+ b2+ c2)Sabc =a2 + b2+ c24R= R(sin2A + sin2B + sin2C) ≤ 9

4RĐẳng thức xảy ra khi ⇔ ∆ABC đều.

b) Từ S = aha= bhb = chc⇔ a1ha= b1hb= c1hc⇒ ∆(a, b, c) ∼ ∆ 1ha,1hb,1hc

⇒ Dấu bằng trong bất đẳng thức (1) xảy ra ∀∆ABCMặt khác áp dụng bất đẳng thức Cauchy ta có:12 hahbhc +hbhcha +hchahb≥ 1233phahbhc

Do đó kết hợp với (1) khi dấu đẳng thức xảy ra ta có:

hacos A + hbcos B + hccos C ≥ 32

3

phahbhc

⇔ (hacos A + hbcos B + hccos C)3 ≥ 27

8 hahbhcDấu đẳng thức xảy ra ⇔ ha = hb = hc⇔ a = b = c ⇔ ∆ABC đềuBài 10

Chứng minh rằng với mọi tam giác ∆ABC và ∆A1B1C1 ta có:cos Asin A1 +cos Bcos B1 +cos Ccos C1 ≤ 12sin A1sin B1sin C1 +sin B1sin A1sin C1 +sin C1sin A1sin B1

Dấu đẳng thức xảy ra ⇔ ∆ABC ∼ ∆A1B1C1Chứng minh

Áp dụng bài toán 2 với:

x = 1sin A1, y =1sin B1, z =1sin C1

Ta có bất đẳng thức cần chứng minh Dấu đẳng thức xảy ra

Trang 19

⇔ ∆ABC ∼ A1B1C1Bài 11

Với hai tam giác ∆ABC và tam giác ∆A1B1C1 bất kì, chứng minh rằng:(b1+ c1) cos A + (c1+ a1) cos B + (a1+ b1) cos C ≤

≤ 12 (b1+ c1)(c1+ a1)a1+ b1 +(c1+ a1)(a1+ b1)b1+ c1 +(b1+ c1)(a1+ b1)c1+ a1

Dấu đẳng thức xảy ra khi nào?Chứng minh

Áp dụng bài toán 2 với:x = b1+ c1y = c1+ a1z = a1 + b1⇔ Ta có bất đẳng thức cần chứng minh.

Mặt khác với a1, b1, c1 là 3 cạnh của ∆A1B1C1, giả sử a1 ≥ b1 ≥ c1 ⇒1

b1+ c1,1c1 + a1,

1

a1+ b1 cũng là 3 cạnh của tam giác Thật vậy ta có:1b1+ c1 ≥ 1c1+ a1 ≥ 1a1+ b1( vì a1 ≥ b1 ≥ c1)Xét 1a1+ b1 ≥ 1b1+ (b1+ c1) ≥ 12(b1+ c1)1a1+ c1 ≥ 1c1+ (b1+ c1) ≥ 12(b1+ c1)⇔ 1a1+ b1 +1a1 + c1 ≥ 1b1+ c1 ⇔ 1b1+ c1,1c1+ a1,1a1+ b1là 3 cạnh của một tam giác Vậy dấu bất đẳng thức xảy ra khi

∆ABC ∼ ∆1b1+ c1,1c1 + a1,1a1+ b1Bài 12

Với A, B, C là ba góc của ∆ABC bất kì, x, y, z là 3 số thực tùy ý, chứng minhrằng:

(−1)n[yz cos nA + xz cos nB + xy cos nC] ≤ 12(x

2+ y2+ z2) (1)

Chứng minh

[x + (−1)n(y cos nC + z cos nB)]2+ (y sin nC − z sin nB)2 ≥ 0

Trang 20

+ 2yz (cos nC cos nB − sin nC sin nB) ≥ 0

mà cos nC cos nB − sin nC sin nB = cos n(B + C) = cos(nπ − nA) = (−1)ncos nAVậy ta có bất đẳng thức:

x2+ y2z2+ 2(−1)n[xy cos nC + yz cos nA + zx cos nB] ≥ 0

(−1)n(yz cos nA + zx cos nBxy cos nC) ≤ 12(x

2

+ y2+ z2)

Ta xét riêng trường hợp x, y, z dương, dấu đằng thức của (1) xảy ra nếu:x = (−1)n+1(y cos nC + z cos nB)y sin nC − z sin nB = 0⇔

y2cos2nC + z2cos2nB + 2yz cos nC cos nB = x2y2cos nC + z2sin2nB − 2yz sin nC sin nB = 0⇔ y2 + z2+ 2yz cos n(B + C) = x2⇔ y2 + z22yz(−1)ncos nA = x2⇔ cos nA = (−1)n+1 y2+ z2− x22yzTương tự:cos nB = (−1)n+1 x2+ z2− y22xzcos nC = (−1)n+1 x2+ y2− z22xy

Điều kiện cần tồn tại ∆ABC là:|y2+ z2− x22yz | ≤ 1|x2+ z2 − y22xz | ≤ 1|x2+ y2− z22xy | ≤ 1

Trang 21

Và ngượi lại nếu chọn x, y, z thỏa mãn bất đẳng thức tam giác (x, y, z > 0) thìta ln tìm được 3 góc A, B, C là 3 góc của tam giác để dấu dẳng thức xảy ra.Bài 13

Chứng minh rằng trong mọi tam giác ∆ABC ta có

(−1)n+1 1xcos nA +1ycos nB +1z cos nC≤ 12 x2+ y2 + z2xyz∀x, y, z dương.Chứng minh

Áp dụng kết quả bài tốn 12 ta có:

(−1)n+1(yz cos nA + zx cos nB + xy cos nC) ≤ 12(x

2

+ y2+ z2)

Chia 2 vế cho xyz > 0 ta có:

(−1)n+1 cos nAx +cos nBy +cos nCz≤ 12 x2+ y2+ z2xyz

Dấu đẳng thức xảy ra khi x, y, z > 0 thỏa mãn bất đẳng thức tam giác.Bài 14

Cho M = 6 cos 4A + 2 cos 4B + 3 cos 4C (với A, B, C là 3 góc của tam giác) Tìm

giá trị bé nhất của M Chứng minh

Áp dụng bài toán 13 với x = 16, y =

12, z =

1

3 (thỏa mãn bất đẳng thức tamgiác) Ta có: (−1)5(6 cos 4A + 2 cos 4B + 3 cos 4C) ≤ 1

2136+ 14 +19161213

⇔ 6 cos 4A + 2 cos 4B + 3 cos 4C ≥ 7Dấu đẳng thức xảy ra nếu:

Trang 22

Vậy minM = −7 khi ∆ABC vuông cân đỉnh B.Bài 15

Cho 2 tam giác bất kì ∆ABC và ∆A0B0C0, chứng minh rằng

M = sin B0sin C0sin 5A + sin C0sin A0cos 5B + sin A0sin B0cos 5C ≤ 98Chứng minh

Áp dụng bài toán 13 với x = sin A0, y = sin B0, z = sin C0 ta có:

M ≤ 1

2(sin

2

A0+ sin2B0 + sin2C0)

Mặt khác trong mọi tam giác ∆A0B0C0 ta dễ có

sin2A0+ sin2B0+ sin2C0 ≤ 98Vậy M ≤ 12.94 =98Bài 16

Với tam giác ∆ABC bất kì, chứng minh rằng:

4 cos 7A − 3 cos 7B + 5 cos 7C ≥ −769120Chứng minh

Áp dụng bài toán 12 với x = 1

4, y = −13, z =15 ta có:(−1)8−1315cos 7A +1415cos 7B −1413cos 7C≤ 12 116+19+125

4 cos 7A − 3 cos 7B + 5 cos 7C ≥ −12 116 +19 +1253.4.54 cos 7A − 3 cos 7B + 5 cos 7C ≥ −769

Trang 23

1.3Sử dụng bất đẳng thức Cauchy chứng minh mộtsố dạng bất đẳng thức xoay vòng

Bài 1

Với a, b, c là các số thực dương, chứng minh rằnga3a2+ b2 + b3b2+ c2 + c3c2+ a2 ≥ a + b + c2Chứng minhTa có:a3a2+ b2 = a(a2+ b2− b2)a2+ b2 = a − b aba2+ b2 ≥ a − ba2+b22a2+ b2 = a − b2Tương tựb3b2+ c2 ≥ b − c2c3c2+ a2 ≥ c − a2

Cộng các bất đẳng thức trên ta thu được bất đẳng thức cần chứng minh.Bài 2Với a, b, c > 0; α, β, γ ≥ 0, chứng minh rằnga a2‘(1 − α)b2a2+ b2+ b b2+ (1 − β)c2b2+ c2+ c c2 + (1 − γ)a2c2+ a2≥≥ (1 − γ2)a + (1 −α2)b + (1 −β2)cChứng minhTa có:a a2 + (1 − α)b2a2+ b2= a1 − αb2a2 + b2= a − αb aba2+ b2 ≥ a − αb2a2+ b2a2+ b2 = a −αb2Tương tự ta thu được:

Trang 25

Chọn α = 1 ta đượcBài 6Với a, b, c > 0, chứng minh rằnga3a2+ b2+ ab +b3b2 + c2+ bc+c3c2+ a2+ ca ≥ a + b + c3Chọn α = 1a ta thu đượcBài 7Với a, b, c > 0, chứng minh rằnga3a2+ b2+ b +ab3a(b2+ c2) + bc +c3c2+ a2+ c ≥ (a + b + c)a1 + 2aChọn α = 1

abc > 0 ta thu đượcBài 8Với a, b, c > 0, chứng minh rằngca3c(a2+ b2) + 1 +ab3a(b2 + c2) + 1 +bc3b(c2+ a2) + 1 ≥ a + b + c1 + 2abcabc

Trang 26

a4a3+ b3 = a(a3+ b3− b3)a3+ b3 = a − ab3a3+ b3 = a − b√b√aa32b32a3 + b3Suy raa4a3+ b3 ≥ a − b√b2√aa3+ b3a3+ b3 ⇔ a4a3b3 + b√b2√a ≥ aTương tựb4b3+ c3 + c√c2√b ≥ bc4c3+ a3 + a√a2√c ≥ c

Cộng các bất đẳng thức trên ta thu được bất đẳng thức cần chứng minh.Bài 11Với a, b, c > 0, chứng minh rằnga5a4+ b4 + b5b4+ c4 + c5c4+ a4 + 12 b2a +c2b +a2c≥ a + b + cChứng minhTa cóa5a4+ b4 = a(a4+ b4− b4)a4+ b4 = a − ab4a4+ b4 = a −b2aa2b2a4+ b4Suy raa5a4+ b4 ≥ a − b22aTương tựa5b4+ c4 ≥ b − c22bc5c4+ a4 ≥ c − a22c

Cộng vế với vế của các bất đẳng thức trên ta thu được bất đẳng thức cần chứngminh.

Trang 27

Với a, b, c > 0, chứng minh rằnga(a3+ b3)a3+ 2b3 +b(b3+ c3)b3+ 2c3 + c(c3+ a3)c3+ 2a3 ≥ 23(a + b + c)Ta cóa(a3 + b3)a3 + 2b3 = a(a3+ 2b3− b3)a3+ 2b3 = a − ab3a3+ 2b3 ≥ a − ba3+b3+b33a3+ 2b3Suy raa(a3+ b3)a3+ 2b3 ≥ a − b3b(b3+ c3)b3+ 2c3 ≥ b − c3c(c3+ a3)c3+ 2a3 ≥ c − a3

Trang 28

c5c2+ a2 ≥ 23c3− a36

Trang 30

Cộng vế với vế các bất đẳng thức trên ta thu được bất đẳng thức được chứngminh.Bài 18Với a, b, c > 0, chứng minh rằngb(b + 2a)a(a + b)2 + c(c + 2b)b(b + c)2 + a(a + 2c)c(c + a)2 ≥ 34 1a +1b +1cChứng minhTa có:1a − 1bab(a + b)2 ≥ 1a − 14b⇔ b(b + 2a)a(a + b)2 ≥ 1a − 14bTương tực(c + 2b)b(b + c)2 ≥ 1b − 14ca(a + 2c)c(c + a)2 ≥ 1c − 14a

Trang 31

c2(c + 2a)

(c + a)2 ≥ c − a4

Cộng vế với vế các bất đẳng thức trên ta thu được bất đẳng thức cần chứngminh.Bài 20Với a, b, c > 0, chứng minh rằngb2ca(ca2cb2+ 1) +c2ab(ab2+ ac2+ 1) +a2bc(bc2+ ba2+ 1) ≥ ab + bc + ca1 + 2abcChứng minhTa cób2a(a2+ b2+ αab) ≥ b2a(a2 + b2+α2(a2+ b2)) =22 + αb2a(a2+ b2)Thu được bất đẳng thứcb2a(a2+ b2+ αab) +c2b(b2+ c2+ αbc) +a2c(c2+ a2αca) ≥ 12 + α 1a +1b +1cChọn α = 1

Trang 32

1.4Bất đẳng thức xoay vòng phân thứcBài 1Với a, b > 1, chứng minh rằng:11 + a +11 + b ≥ 21 +√abChứng minhBất đẳng thức tương với:(a + b) + 21 + (a + b) + ab ≥ 21 +√ab⇔ (a + b) + 2 + (a + b)√ab + 2√ab ≥ 2 + 2(a + b) + 2ab⇔ (a + b)(√ab − 1) + 2√ab(1 −√ab) ≥ 0⇔ (√ab − 1)(√a −√b)2 ≥ 0 (Hiển nhiên vì √ab > 1 )Bài 2Với 0 < a, b < 1, chứng minh rằng:11 + a +11 + b ≤ 21 +√abChứng minh

Trang 33

Ta thu được:1(1 + a)n + 1(1 + b)n ≥ 2(11+a +1+b12 )n

Áp dụng kết quả bài 1 ta thu được.1(1 + a)n + 1(1 + b)n ≥ 2( 11 +√ab)n = 2(1 + ab)nBài 4Với 0 < a, b < 1, chứng minh rằng:1n√1 + a +1n√1 + b ≤ pn 21 +√abChứng minhÁp dụng bất đẳng thứcn√a + √nb2 ≤ nra + b2 Với a, b > 0Ta thu được1n√1 + a+1n√1 + b ≤ ns11+a + 11+b2Áp dụng kết quả ở ví dụ 2 ta thu được

1n√1 + a +1n√1 + b ≤ 2ns11 +√ab =2np1 +√abBài 5Với a, b > 1 chứng minh rằnga1 + b +b1 + a ≥ 2√ab1 +√abChứng minh

Trang 34

⇔ (a + b + 1)( 11 + b+11 + a) ≥ (1 + 2√ab) 21 +√abTa cóa + b + 1 ≥ 1 + 2√ab11 + b +11 + a ≥ 21 +√ab

Nhân hai vế hai bất đẳng thức trên ta thu được bất đẳng thức cần chứng minh.Bài 6Với a, b, c > 0, chứng minh rằng11 + a+11 + b +11 + c ≥ 31 +√3abcChứng minh

Bất đẳng thức đã cho tương đương với

P = 11 + a+11 + b +11 + c+11 +√3abc ≥ 41 +√3abcTa cóP ≥ 21 +√ab +21 +pc√3abc≥ 41 +p4 abc√3abc= 41 +√3abcBài 7Với a, b, c > 1, chứng minh rằng2 + b + c1 + a +2 + c + a1 + b +1 + a + b1 + c ≥ 6Chứng minh

Trang 35

Ta có3 + a + b + c ≥ 3(1 +√3abc)11 + a+11 + b +11 + c ≥ 31 +√3abc

Nhân vế với vế của hai bất đẳng thức trên chúng ta thu được bất đẳng thức cầnchứng minh.Bài 8Với a, b, c > 1, chứng minh rằng1(1 + a)3 + 1(1 + b)3 + 1(1 + c)3 ≥ 3(1 +√3abc)3Chứng minh

Bất đẳng thức đã cho tương đương với

P = 1(1 + a)3 + 1(1 + b)3 + 1(1 + c)3 + 1(1 +√3abc)3 ≥ 4(1 +√3abc)3Áp dụng kết quả bài 3 ta cóP ≥ 2(1 +√ab)3 + 2(1 +pc√3abc)3≥ 4(1 +p4 abc√3abc)3≥ 4(1 +√3abc)3Bài 9Với 0 < a, b, c < 1, chứng minh rằng1√1 + a +1√1 + b +1√1 + c ≤ p 31 +√3abcChứng minh

Bất đẳng thức đã cho tương đương với

Trang 36

Áp dụng kết quả bài 4 ta thu đượcP ≤ p 21 +√ab+q 21 +pc√3abc≤ q 41 +p4 abc√3abc= p 41 +√3abcBài 10Với a, b, c > 1, chứng minh rằngP = (2 + b + c1 + a )2+ (2 + c + a1 + b )2+ (2 + a + b1 + c )2 ≥ 12Ta cóP ≥ (2+b+c

1+a +2+c+a1+b + 2+a+b1+c

3 )2 ≥ 3(63)2 = 12Áp dụng ví dụ 7Bài 11

Với a, b, c là các số thực dương, chứng minh rằnga5b2 + b5c2 + c5a2 ≥ a3+ b3+ c3Chứng minhTa cóa5b2 + ab2 ≥ 2a3b5c2 + bc2 ≥ 2b3c5a2 + ca2 ≥ 2c3a3+ b3+ c3 ≥ ab2+ bc2+ ca2

Cộng 4 bất đẳng thức trên chúng ta thu được bất đẳng thức cần chứng minh.Bài 12

Trang 37

a5bc + abc ≥ 2a3b5ca + abc ≥ 2b3c5ab + abc ≥ 2c3a3+ b3+ c3 ≥ 3abc

Cộng 4 bất đẳng thức trên chúng ta thu được bất đẳng thức cần chứng minh.Bài 13

Với a, b, c là các số thực dương chứng minh rằnga5b3 +b5c3 + c5a3 ≥ a3b +b3c +c3aChứng minhTa có: a5b3 + ab ≥ 2a3bSuy ra: a5b5 + 2ab ≥ a3b +a3b + ab ≥a3b + 2a2Tương tự:b5c3 + 2bc ≥ b3c + 2b2c5a3 + 2ca ≥ c3a + 2c2

mà 2(a2+ b2+ c2) ≥ 2(ab + bc + ca)

Cộng 4 bất đẳng thức trên chúng ta thu được bất đẳng thức cần chứng minh.Bài 14

Với a, b, c là các số thực dương chứng minh rằnga3a + 2b +b3b + 2c +c3c + 2a ≥ 13(a2+ b2+ c2)Chứng minhTa có:9a3a + 2b + a(a + 2b) ≥ 6a29b3b + 2c + b(b + 2c) ≥ 6b29c3c + 2a + c(c + 2a) ≥ 6c2

mà 2(a2+ b2+ c2) ≥ 2(ab + bc + ca)

Trang 38

Bài 15

Với a, b, c là các số thực dương chứng minh rằnga3(b + c)2 + b3(c + a)2 + c3(a + b)2 ≥ 14(a + b + c)Chứng minhTa có:8a3(b + c)2 + (b + c) + (b + c) ≥ 6a8b3(c + a)2 + (c + a) + (c + a) ≥ 6b8c3(a + b)2 + (a + b) + (a + b) ≥ 6c

Cộng 3 bất đẳng thức trên ta thu được bất đẳng thức cần chứng minh.Bài 16

Với a, b, c là các số thực dương chứng minh rằnga3b(c + a)+b3c(a + b)+c3a(b + c) ≥ 12(a + b + c)Chứng minhTa có:4a3b(c + a)+ 2b + (c + a) ≥ 6a4b3c(a + b)+ 2c + (a + b) ≥ 6b4c3a(b + c)+ 2a + (b + c) ≥ 6c

Cộng 3 bất đẳng thức trên ta thu được bất đẳng thức cần chứng minh.Bài 17

Trang 39

b4

ca2 + c + a + a ≥ 4bc4

ab2 + a + b + b ≥ 4c

Cộng 3 bất đẳng thức trên ta thu được bất đẳng thức cần chứng minh.Bài 18

Với a, b, c là các số thực dương chứng minh rằnga3(a + b)(b + c) +b3(b + c)(c + a) +c3(c + a)(a + b) ≥ 14(a + b + c)Chứng minhTa có:8a3(a + b)(b + c) + (a + b) + (b + c) ≥ 6a8b3(b + c)(c + a) + (b + c) + (c + a) ≥ 6b8c3(c + a)(a + b) + (c + a) + (a + b) ≥ 6c

Cộng 3 bất đẳng thức trên ta thu được bất đẳng thức cần chứng minh.Bài 19

Với a, b, c là các số thực dương chứng minh rằnga2b3 + b2c3 + c2a3 ≥ 9a + b + cChứng minhTa có bất đẳng thứcb3a2 +c3b2 +a3c2 ≥ a + b + cSuy ra:a2b3 + b2c3 + c2a3 = (1b)3(1a)2 + (1c)3(1b)2 + (1a)3(1c)2 ≥ 1a +1b +1c ≥ 9a + b + cBài 20

Ngày đăng: 07/07/2023, 15:40